LSAT and Law School Admissions Forum

Get expert LSAT preparation and law school admissions advice from PowerScore Test Preparation.

 Administrator
PowerScore Staff
  • PowerScore Staff
  • Posts: 8917
  • Joined: Feb 02, 2011
|
#23684
Complete Question Explanation

Must be True. The correct answer choice is (D)

The stimulus argues that complaints against the price mark-up for milk are most frequent when they are the least justified, since those complaints occur when increasing wholesale prices force up the price to consumers. The argument is based on the idea that since those price increases simply pass the cost directly to the consumer, at that point the price difference is proportionally the least.

In the numbers, the argument makes sense. Basically, it is something like this:

  • ..... ..... Wholesale ..... Retail ..... % Markup

    Lowest ..... $1.00 ..... $2.00 ..... 100%

    Highest ..... $2.00 ..... $3.00 ..... 50%
Realistically, the argument makes several assumptions such as that the percentage markup is the only thing that could justify complaints; however, you are simply asked to identify what must be true, so you should not worry too much about any logical flaws in the stimulus.

Answer choice (A): The argument is that the percent markup rises, not that the actual retail price rises when wholesale prices fall, so this choice is wrong.

Answer choice (B): This choice does not make sense, since the argument is in part based on the premise that dairy farmers raise their prices from time to time, which may be the result of passing costs along. This response is incorrect.

Answer choice (C): Since the stimulus gave you no information about "extended periods," you should not select this incorrect choice. There is no stated reason to suppose that simply because a markup is higher when prices are low, the markup would continue to increase as the price holds.

Answer choice (D): This is the correct answer choice. The argument is that the markups are proportionally the greatest when prices are dropping, which means that the prices are not decreased proportionally when prices are dropping. Mathematically, a proportional reduction would mean that [Retail/Wholesale] holds the same ratio for the high and the low prices. The passage states that does not occur.

Answer choice (E): You should not confuse the dairy farmers with the milk bottlers. Since you do not know anything about the dairy farmers, other than that they sometimes increase their prices, you cannot tell when they are making the smallest profit. Furthermore, you cannot even be certain that the milk bottlers make the lowest absolute profit when prices are high, only that the profit margin is the lowest when prices are high.
 Mastering_LSAT
  • Posts: 35
  • Joined: Jul 30, 2020
|
#95905
Hello PowerScore,

I was wondering if it is correct on my part to take the following approach with the incorrect answer choice (C). Would greatly appreciate your feedback!
You do not have the required permissions to view the files attached to this post.
 Rachael Wilkenfeld
PowerScore Staff
  • PowerScore Staff
  • Posts: 1358
  • Joined: Dec 15, 2011
|
#95994
Hi Mastering,

I agree with you overall. I would say there are many reasons to eliminate answer choice (C) here, and any one of them would be enough to knock it out of contention. In order, I would probably eliminate it because of the "extended period" language, then due to the certainty of the language in the answer choice that does not match the certainty of the language in the stimulus, and finally because of the ambiguity with the amount of markup v percent of markup. But again, and of the reasons would be enough to knock that answer choice out of contention.

Great work!
 Mastering_LSAT
  • Posts: 35
  • Joined: Jul 30, 2020
|
#96051
Thank you, Rachael!
User avatar
 ericsilvagomez
  • Posts: 49
  • Joined: Oct 16, 2023
|
#104276
Hi,

Can you elaborate on how you included the numbers when solving the problem? It's the first time I have encountered it on LR. Also, I selected A, but it was more of a guess because the discussion about the different prices confused me.
 Robert Carroll
PowerScore Staff
  • PowerScore Staff
  • Posts: 1787
  • Joined: Dec 06, 2013
|
#104331
ericsilvagomez,

The numbers are invented to make clear what the stimulus is saying. Modeling a numerical situation with actual numbers can be a useful way to visualize the consequences of the stimulus, which we need to do for a Must Be True question. Obviously, the specific numbers chosen are irrelevant - we just need to make sure that the markup is lower when the raw milk price is higher, and higher when it's lower.

Robert Carroll

Get the most out of your LSAT Prep Plus subscription.

Analyze and track your performance with our Testing and Analytics Package.